Download as pdf or txt
Download as pdf or txt
You are on page 1of 14

3H Electromagnetism 2008-2009 Solutions 4 – mostly Faraday and Maxwell page 1

40 A particle with mass m and charge q is moving with velocity v in a uniform magnetic field B.
The electric field E is zero. Write down the Lorentz force law for this case, and show that |v|2
and v · B are constants of the motion.

Solution: With E zero, the Lorentz force law gives the force on the particle to be F = qc v ∧ B .
Using Newton’s law, the acceleration a is
dv q
a= = v ∧ B.
dt mc
Then
d dv 2q
(v · v) = 2v · = v · (v ∧ B) = 0
dt dt mc
since v and v ∧ B are perpendicular (note, this result holds even if B is non-uniform). Likewise,

d dv q
(v · B) = ·B= (v ∧ B) · B = 0
dt dt mc
where the first equality holds since B is uniform (and constant in time).

41 Particles of mass M and charge q in the region y < 0 accelerate in the ey direction through
appotential difference, or voltage, V . If they start from rest, show that their final velocity is
2qV /M ey . The particles then pass through a slit S into the region y > 0 , where there is a
uniform magnetic field B = B ez , and zero electric field. A photographic plate is placed beside
the slit, in the y = 0 plane, as shown below.


Show that the particles hit the plate at a distance from S which is proportional to M , and find
the constant of proportionality. This is the principle of the mass spectrometer.
[Hint: first observe that the particles travel in a circle in the region y > 0, and then find its
radius, making use of the formula a = v 2 /R for the acceleration towards the centre of a particle
travelling with speed v round a circle of radius R.]

Solution: The (electrostatic) potential difference being V , the particles pick up an energy V q
in the region y < 0. This must equal the kinetic energy 21 M v 2 , if the particles start from rest
and finish with a velocity v, where v = |v|. Hence
1
V q = M v2
2
p
or v = 2qV /M . Since the p acceleration is in the ey direction, this must also be the direction
of the final velocity and v = 2qV /M ey , as required.
In the region y > 0 , the magnetic field produces a force qc v ∧ B on the particles, in the xy
plane and perpendicular to their velocity. As in question 40, the magnitude of the velocity is
constant, and, since v and B remain at right angles, the magnitude of the force is F = qvB/c.
This gives the particles a constant acceleration at right angles to their motion, so they move in
a circular trajectory. To find the radius of the circle, use the fact that a particle moving with
3H Electromagnetism 2008-2009 Solutions 4 – mostly Faraday and Maxwell page 2

speed v on a circle of radius R accelerates at a rate a = v 2 /R towards the centre. Equating this
to F/M , we have
v2 qvB
=
R cM

or R = cM v/(qB). Putting in the value of v found in the first part, R = c √2VM and so the
√ qB
distance d = 2R at√which the particles hit the plate is indeed proportional to M , with constant

of proportionality 8V c/( qB) .

42 If C is a closed curve and S a spanning surface, show that


I Z
r ∧ dr = 2 n dS .
C S

[Hint: use the component form of Stokes’ theorem.] Deduce that an alternative expression for
the magnetic moment of a closed loop C carrying a current I is
I
I
µ= r ∧ dr .
2c C

Solution: Using the component form of Stokes’ theorem with ψ = εijk xj , where (x1 , x2 , x3 ) = r,


I Z
εijk xj dxk = εklm nl εijk xj dS
C S ∂xm
Z
= nl δjm (δil δjm − δim δjl ) dS
S
Z Z
= nl (3δil − δij δjl ) dS = 2 ni dS
S S

which is equivalent to the quoted result. The alternative expression for the magnetic moment
now follows on substituting into the formula for µ given in lectures.

43 In lectures, the magnetic dipole moment of a closed current-carrying circuit was discussed. If
instead we have a localised current distribution, the relevant formula is
1
Z
µ= r ∧ j(r) dV .
2c

(Compare this with the formula obtained in question 42.) Use this result to compute the magnetic
dipole moment of a sphere of radius a carring a charge Q uniformly distributed on its surface
and rotating with constant angular velocity ω about a fixed axis.
3H Electromagnetism 2008-2009 Solutions 4 – mostly Faraday and Maxwell page 3

Solution: Choose the axis of rotation in the z-direction, ω = ωez . Then, by symmetry, the
dipole moment is in the z-direction too, µ = µez , with
1
Z
µ= ez · (r ∧ j) dV .
2c
We have a surface charge distribution and, using spherical polar coordinates,

surface area of sphere: S = 4πa2 ,


Q
surface charge density: ρ= ,
4πa2
surface velocity: v = ωa sin θ eφ ,

surface current: j = ρv = sin θ eφ .
4πa
(Remember the formula j = ρv from term 1.) Then
 

ez · (r ∧ j) = ez · a er ∧ sin θ eφ
4πa

= sin θ ez · (−eθ )


= sin2 θ .

Since the charge is on the surface of the sphere, we need only do a surface integral to find µ.
This integral is

1 π
Z 2π

Z
µ = a dθ a sin θ dφ sin2 θ
2c 0 0 4π
2 Z π
Qa ω
= sin3 θ dθ
4c 0

Qa2 ω
=
3c
and so
Qa2 ω Qa2
µ= ez = ω.
3c 3c

44 Given that the vector potential for an ideal magnetic dipole is A(r) = µ ∧ r/|r|3 , show that the
corresponding magnetic field for r 6= 0 is

3r(µ · r) − µ(r · r)
B(r) =
|r|5

(Challenge: argue that the result valid for all r is B(r) = (3r(µ · r) − µ(r · r))/|r|5 + 8π
3
µ δ (3) (r). )
3H Electromagnetism 2008-2009 Solutions 4 – mostly Faraday and Maxwell page 4

Solution: We can use the vector calculus result ∇ ∧ (k ∧ C) = k ∇ · C − k · ∇C for constant


k to see that B is given by
   
r r
B = µ∇ · −µ·∇ .
|r|3 |r|3

Now ∇ · (r/|r|3 ) = 0 for r 6= 0 (this is a result we already used a fair bit in electrostatics) while
the ith component of the second term is
   
∂ xi δij 3 2xi xj
−µj = −µj +
∂xj (xk xk )3/2 (xk xk )3/2 2 (xk xk )5/2
µi 3xi µj xj
= − 3/2
+
(xk xk ) (xk xk )5/2
from which the result follows almost immediately. To get a result valid for all r, first recall
from electrostatics that ∇ · (r/|r|3 ) = 4π δ (3) (0) ; for the rest, you’ll need to use the same style
of argument as in electrostatics, and the component form of the divergence theorem.

45 A solenoid with N turns per unit length has radius R and length l, and carries a current I.
Show that the total magnetic flux through a single loop of the solenoid is equal to 4π 2 R2 IN/c ,
and hence that its self-inductance L is

4π 2 R2 2
L= N l.
c2

Solution: Suppose for definiteness that the axis of the solenoid lies along the z axis. Then (cf
equation (2.4.7) of your notes) the magnetic field is equal to (4πIN/c) ez inside the solenoid,
and zero outside. The cross-sectional area of the solenoid is πR2 , so the total magnetic flux ΦB
through a single loop of the coil is ΦB = 4π 2 R2 IN/c . To calculate the self-inductance we need
the total flux enclosed by the circuit, which loops a total of N l times round the solenoid; hence
the flux through the circuit is equal to Φtot = N lΦB = 4π 2 R2 N 2 l I/c . The self-inductance L
is defined by Φtot = cLI (equation (3.2.4) of your notes), and so the formula quoted in the
question follows immediately.

46 An infinite straight solenoid of radius R is encircled by a single loop C2 . Show that the M12 ,
the mutual inductance of the loop and the solenoid, is given by

4π 2 R2
M12 = N.
c2

Solution: This case is actually easier than the last one. Suppose the current round the solenoid
is I1 ; then the field inside it is 4πI1 N/c while that outside it is 0. The area of the region where
the field is non-zero is πR2 , so the total flux through C2 is Φ12 = 4π 2 R2 I1 N/c . The mutual
inductance is defined via Φ12 = cM12 I1 , equation (3.2.1) in notes, and so the quoted formula is
confirmed.
3H Electromagnetism 2008-2009 Solutions 4 – mostly Faraday and Maxwell page 5

47 Calculate the mutual inductance M12 between a wire in the form of an ellipse of axes 2a and 2b
(a > b), and an infinite straight wire lying in the plane of the ellipse parallel to its minor axis
and at a distance d (> a) from its centre.
You may use Z π
dφ π
=p for |α| > |β| .
0 α + β cos φ α2 − β 2

Solution: The flux through the ellipse arising from a current I along the wire is
Z
Φ = cM12 I = B · n dS.
S

The magnetic field is


2I 1
B=
c d+x
and thus
2 dS
Z
M12 = 2 .
c S d+x
To do the integral over the ellipse we switch to elliptical polar coordinates

x = ar cos φ, y = br cos φ, dx dy = abr dr dφ.

Then
2ab 1 π r
Z Z
M12 = 2
dφ dr
c 0 −π d + ar cos φ
4ab 1 πr
Z
= 2
√ dr
c 0 d2 − a2 r2
 √ 2 1
4πab d − a2 r 2
= −
c2 a2 0
4πb  √ 
= d − d2 − a2 .
ac2

48 A circular loop of wire of radius a lies in the xy-plane with its centre on the z-axis. Find the
induced emf in the wire due to a variable magnetic field B = B0 sin(ωt), where B0 is a constant
vector which makes an angle α with the z-axis and ω is a constant. Once any transient effects
have died down, this will induce an alternating current I(t) = I0 sin(ωt + ϕ) in the wire. Find
the amplitude I0 if the resistance of the wire is R and its self-inductance is L.

Solution: The induced emf is given by


1 dΦ
E =−
c dt
where Φ is the flux linking the wire loop,
Z
Φ= B · n dS = πa2 B0 cos α sin ωt.
S
3H Electromagnetism 2008-2009 Solutions 4 – mostly Faraday and Maxwell page 6

Hence
πa2
E =− ωB0 cos α cos ωt.
c
The external emf E induces a current I in the wire which in turn produces a magnetic field
inducing an emf LI˙ counteracting the change in current. Hence
RI = Etotal = E − LI˙ ⇒ LI˙ + RI = E.
The general solution of the homogeneous equation is an exponentially decaying current, the
transient current induced by switching the external field on. We are interested in the steady
state current which is the particular solution of form
I(t) = I0 sin(ωt + φ)
where I0 is the amplitude and φ is a possible phase shift. Substituting into the differential
equation and separating the sine and cosine components we get
I = I0 sin(ωt + φ) = I0 cos φ sin ωt + I0 sin φ cos ωt,
I˙ = −I0 cos(ωt + φ) = −I0 ω sin φ sin ωt + I0 ω cos φ cos ωt,

πa2
LωI0 cos φ + RI0 sin φ = − ωB0 cos α,
c
RI0 cos φ − LωI0 sin φ = 0.
Then
R πa2
I0 sin φ = − ωB0 cos α,
R 2 + L2 ω 2 c
Lω πa2
I0 cos φ = − 2 ωB0 cos α
R + L2 ω 2 c
and thus the amplitude is
p πa2 ωB0 cos α
I0 = (I0 sin φ)2 + (I0 cos φ)2 = √ .
c R 2 + L2 ω 2

2
49 Verify that a gauge transformation which satisfies ∇2 Λ− c12 ∂t

2 Λ = 0 preserves the Lorentz gauge

condition.

Solution: Under a gauge transformation parametrised by Λ(x, t) we have


1 ∂Λ
(A, φ) → (A′ , φ′ ) = (A + ∇Λ, φ − )
c ∂t
and so
1 ∂φ′ 1∂ 1 ∂Λ 
∇ · A′ + = ∇ · (A + ∇Λ) + φ−
c ∂t c ∂t c ∂t
1 ∂φ 1 ∂2Λ
= ∇·A+ + ∇2 Λ −
c ∂t c ∂t2
1 ∂φ
= ∇·A+
c ∂t
3H Electromagnetism 2008-2009 Solutions 4 – mostly Faraday and Maxwell page 7

using the given property of Λ. Thus, if (A, φ) satisfies the Lorentz gauge condition ∇·A+ 1c ∂φ
∂t
=
0, then so does (A′ , φ′ ).

50 Show that if A satisfies the equations

1 ∂2A
∇ · A = 0, ∇2 A =
c2 ∂t2
then E and B defined as
1 ∂A
E=− and B = ∇ ∧ A
c ∂t
satisfy Maxwell’s equations in empty space. Show that a possible form for A is
   
2π 2π
A = a cos (z − ct) ex + a sin (z − ct) ey ,
λ λ

where a and λ are constants. Find E and B and show that they rotate about the z-axis with
frequency c/λ.

Solution: First, check that Maxwell’s equations for empty space hold if A satisfies the two
conditions given in the question:

∇ · B = ∇ · (∇ ∧ A) = 0,
1∂
∇·E = − (∇ · A) = 0,
c ∂t
1 ∂E 1 ∂2A
∇∧B− = ∇(∇ · A) − ∇2 A + 2 2 = 0,
c ∂t c ∂t
1 ∂B 1 ∂A 1 ∂
∇∧E+ = − ∇∧ + (∇ ∧ A) = 0.
c ∂t c ∂t c ∂t
Next check the given vector potential satisfies the conditions:
∂Ax ∂Ay
∇·A = + = 0,
∂x ∂y
 2
2 ∂2A 2π 1 ∂2A
∇A = = − A = .
∂z 2 λ c2 ∂t2

The fields are:


     
1 ∂A 2πa 2π 2π
E = − = − sin (z − ct) ex + cos (z − ct) ey
c ∂t λ λ λ
     
2πa 2π 2π
B = ∇∧A= − cos (z − ct) ex − sin (z − ct) ey
λ λ λ

Looking down the z-axis this represents the field vectors rotating clockwise with period λ/c and
thus frequency c/λ; also B is a quarter turn behind E.
3H Electromagnetism 2008-2009 Solutions 4 – mostly Faraday and Maxwell page 8

51 Show that Maxwell’s equations for empty space are satisfied by


1 ∂A
E=− , B=∇∧A
c ∂t
so long as the vector potential A satisfies the wave equation
1 ∂ 2A
∇2 A − =0
c2 ∂t2
and the gauge condition ∇ · A = 0.
Confirm that a solution of this wave equation is given by
 
ψ(r − ct)
A=∇∧ k ,
r
where r = |x|, k is a fixed vector and ψ(r − ct) is an arbitrary twice-differentiable function.

Solution: First, check that Maxwell’s equations for empty space hold if A satisfies the two
conditions given in the question:
∇ · B = ∇ · (∇ ∧ A) = 0,
1∂
∇·E = − (∇ · A) = 0,
c ∂t
1 ∂E 1 ∂2A
∇∧B− = ∇(∇ · A) − ∇2 A + 2 2 = 0,
c ∂t c ∂t
1 ∂B 1 ∂A 1 ∂
∇∧E+ = − ∇∧ + (∇ ∧ A) = 0.
c ∂t c ∂t c ∂t
For the second part of the question, the calculation goes as follows. First,
  
2 2 ψ(r−ct)
∇ A = ∇ ∧ k∇
r
1 d2
  
ψ(r−ct)
= ∇∧ k r
r dr2 r
ψ ′′ (r−ct)
 
= ∇∧ k
r
1 d2
(using the result ∇2 f (r) = r dr2
(rf (r)) , which can be deduced easily from the vector calculus
handout). Next,
∂2A ∂ 2 ψ(r−ct)
  
= ∇∧ k 2
∂t2 ∂t r
ψ ′′ (r−ct)
 
2
= c ∇∧ k .
r
Hence,
1 ∂ 2A
∇2 A − =0
c2 ∂t2
as required.
Note: that the given form of the potential satisfies the gauge condition follows simply from the
fact that ∇ · ∇ ∧ V = 0 for any vector field V.
3H Electromagnetism 2008-2009 Solutions 4 – mostly Faraday and Maxwell page 9

52 Show that the real fields E(x, t) and B(x, t) determined by

E + iB = φ(x + iy)f (z − ct)(ex + iey )

satisfy Maxwell’s equations in a source-free region of space-time if φ is an analytic function of


x + iy and f is differentiable but not necessarily real.

Solution: Maxwell’s equations in empty space are

1 ∂B
∇ · E = 0, ∇∧E+ c ∂t
= 0,
1 ∂E
∇ · B = 0, ∇∧B− c ∂t
= 0.

In terms of
F = E + iB = φ(x + iy)f (z − ct)(ex + iey )
these can be written as
i ∂F
∇ · F = 0, ∇∧F− = 0.
c ∂t
Check these equations:
 
∂φ ∂φ
∇·F = +i f (z − ct) = 0 since φ is analytic,
∂x ∂y
 
i ∂F ∂φ ∂φ
∇∧F− = i +i f (z − ct)ez
c ∂t ∂x ∂y
∂f (z − ct)
−iφ(x + iy) (ex + iey )
∂z
i ∂f (z − ct)
− φ(x + iy) (ex + iey )
c ∂t
= 0.

53 Sources ρ(x, t) and j(x, t) are zero outside a given sphere about the origin for all time. Show
that a solution of Maxwell’s equations in the region outside the sphere is given by
1
E(x, t) = − x ∧ ∇Ż(x, t), B(x, t) = ∇ ∧ (x ∧ ∇Z(x, t)),
c
2 f (r−ct)
provided Z satisfies (∇2 − c12 ∂t

2 )Z = 0 there. Verify that Z = r
satisfies this equation in
the region r > 0 for an arbitrary function f , where r = |x|.

Solution: Note −x ∧ ∇Z = ∇ ∧ (xZ) ≡ A [can you see why the first equality holds?] and
thus
1 ∂A
E= , B = ∇ ∧ A.
c ∂t
Proceed as in question 51 to show that Maxwell’s equations are satisfied provided A satisfies
the wave equation and has vanishing divergence,

∇ · A = ∇ · (∇ ∧ (xZ)) = 0.
3H Electromagnetism 2008-2009 Solutions 4 – mostly Faraday and Maxwell page 10

Verify wave equation for given Z:


1 ∂2 f ′′ (r − ct) 1 ∂2
∇2 Z = (rZ) = = Z.
r ∂r2 r c2 ∂t2

54 A circuit consists of a long, straight wire of radius a carrying a current I and a cylindrical shell
of inner radius b and outer radius d, coaxial with the wire, carrying the return current. Calculate
the energy density of the magnetic field and the magnetic energy per unit length. [Hint: use
Ampère’s Law to calculate the magnetic field.] Given that the total magnetic energy W can be
expressed in terms of the self-inductance L of the circuit and the current I as
1
W = LI 2 ,
2
show that the self-inductance per unit length of the coaxial line is
d4 3d2 − b2
  
2 1 b d
L= 2 + ln + ln − .
c 4 a (d2 − b2 )2 b 4(d2 − b2 )

Solution: Using cylindrical coordinates, by symmetry, the magnetic field is azimuthal and a
function of the radius only, B(r) = B(ρ)eφ . Ampère’s Law for a circle of radius ρ concentric
with the cylinders is

I
B(r) · dr = 2πρB(ρ) = Iρ ,
C c
where Iρ is the current flowing through the disk of radius ρ. This determines the magnetic field
as 



 ca2
I for ρ < a,

 I 2
for a < ρ < b,

cρ 
B= 2 ρ 2 −b2



 cρ
1 − d2 −b2 I for b < ρ < d,


0 for d < ρ.

The energy density is



ρ2


 2πc2 a4
I2 for ρ < a,

 1
I2 for a < ρ < b,

1 2 
2πc2 ρ2
u= B = 
d2 −ρ2 2

8π  1
I 2 for b < ρ < d,


 2πc2 ρ2 d2 −b2

0 for d < ρ.

The total electromagnetic energy per unit length is the integral of the energy density
Z ∞ Z 2π
W = u ρ dφ dρ
0 0
Z a 2
I2 I2 b 1 I 2 d 1 d2 − ρ2
Z Z 
3
= 2 4 ρ dρ + 2 dρ + 2 dρ
ca 0 c a ρ c b ρ d2 − b2
I2 1
  
b 1 2 b 2 2 2 1 4 4
= 2 + ln + 2 d ln − d (d − b ) + (d − b )
c 4 a (d − b2 )2 a 4
2 4 2 2
  
I 1 b d d 3d − b
= 2 + ln + ln − .
c 4 a (d2 − b2 )2 b 4(d2 − b2 )
3H Electromagnetism 2008-2009 Solutions 4 – mostly Faraday and Maxwell page 11

This gives the required expression for the self-inductance L.

55 Show that Maxwell’s equations for empty space are invariant under the transformation

E → E′ = aE + bB, B → B′ = −bE + aB,

where a and b are constants. Under what condition are the energy density (E2 + B2 )/(8π) and
the Poynting vector (c/4π)E ∧ B invariant?

Solution: Substitute into Maxwell’s equations:

∇ · B′ = a∇ · B − b∇ · E = 0,
∇ · E′ = a∇ · E + b∇ · B = 0,
1 ∂E′
   
1 ∂B 1 ∂E
∇ ∧ B′ − = −b ∇ ∧ E + +a ∇∧B− = 0,
c ∂t c ∂t c ∂t
1 ∂B′
   
1 ∂B 1 ∂E
∇ ∧ E′ + = a ∇∧E+ +b ∇∧B− = 0.
c ∂t c ∂t c ∂t

Transform energy density and Poynting vector:


1 1 2
u′ = (E′2 + B′2 ) = (a + b2 )(E2 + B2 ) = (a2 + b2 )u,
8π 8π
c ′ c
S′ = E ∧ B′ = (aE + bB) ∧ (−bE + aB)
4π 4π
c 2
= (a E ∧ B − b2 B ∧ E) = (a2 + b2 )S.

So energy density and Poynting vector are invariant provided a2 + b2 = 1.

56 A long, straight wire of radius a and resistance per unit length R carries a current I. Calculate
the Poynting vector both inside and outside the wire. Explain how your result relates to the
energy dissipated in the wire.

Solution: Suppose the wire is running in the z direction. From Ohm’s Law the current I is
related to the electric field in the direction of current flow as

E = RI ez ,

where R is the resistance per unit length. This field is tangential to the interface and thus
continuous there, so the field has the same uniform value both inside and outside, at least close
to the wire.
The magnetic field can be calculated by Ampère’s Law, as in lectures:

 2ρ I e for ρ < a,
ca2 φ
B(ρ, φ, z) =
 2 I eφ for a < ρ.

3H Electromagnetism 2008-2009 Solutions 4 – mostly Faraday and Maxwell page 12

The Poynting vector, inside and close to the wire, is



c  −RI 2 ρ e for ρ < a,
2πa2 ρ
S= E∧B=
4π  −RI 1 eρ for a < ρ.
2
2πρ

The conservation law for static fields is

−∇ · S = j · E,

where the left hand side is the local inflow of energy while the right hand side is the work done
on the charge carriers. Outside we have
1 ∂
−∇ · S = − (ρSρ ) = 0 = j · E.
ρ ∂ρ
The Poynting vector is divergence free and there are no charge carriers, so no energy is dissipated.
Inside the wire
1 ∂ 1
−∇ · S = − (ρSρ ) = RI 2 2 = j · E,
ρ ∂ρ πa
where j = πaI 2 ez is the current density inside the wire. The work done on the charge carriers is
RI 2 per unit length of the wire. This is the energy dissipated as heat in the wire.

57 Show that the ‘generalised’ Maxwell equations [MM1. . . MM4] imply the continuity equation
∇ · jm + ∂ρ∂tm = 0 for the magnetic current and charge density.

Solution: [MM3] reads


1 ∂B 4π
∇∧E=− − jm .
c ∂t c
Take the divergence of this and use ∇ · (∇ ∧ E) = 0 to find

1∂ 4π
0=− ∇·B− ∇ · jm .
c ∂t c
By [MM2], ∇ · B = 4πρm and so, cancelling common factors,

∂ρm
0= + ∇ · jm
∂t
which is the desired result.

58 Verify that, in an electromagnetic field independent of the coordinate y, the vectors


∂S ∂S 1 ∂S
E= ex − ez , B=− ey
∂z ∂x c ∂t
satisfy Maxwell’s equations in empty space if

∂ 2S ∂ 2S 1 ∂2S
+ = .
∂x2 ∂z 2 c2 ∂t2
3H Electromagnetism 2008-2009 Solutions 4 – mostly Faraday and Maxwell page 13

By considering solutions of the form

S = e−iωt [K cos(αx) + L sin(αx)][N cos(βz) + M sin(βz)],

where K, L, M, N, α, β and ω are constants, find an electromagnetic field in the region of space
0 ≤ x ≤ a, 0 ≤ z ≤ b such that the tangential component of the electric field vanishes over the
planes z = 0, z = b, x = 0 and x = b. Show also that
 ω 2  rπ 2  sπ 2
= + ,
c a b
where r, s = 1, 2, 3, . . .

Solution: Maxwell’s equations in empty space:


 
∂ 1 ∂S
∇·B = − = 0,
∂y c ∂t
   
∂ ∂S ∂ ∂S
∇·E = + − = 0,
∂x ∂z ∂z ∂x
     
1 ∂E ∂ 1 ∂S ∂ 1 ∂S
∇∧B− = − − ex + − ez
c ∂t ∂z c ∂t ∂x c ∂t
 
1 ∂ ∂S ∂S
− ex − ez = 0,
c ∂t ∂z ∂x
      
1 ∂B ∂ ∂S ∂ ∂S 1∂ 1 ∂S
∇∧E+ = − − ey + − ey
c ∂t ∂z ∂z ∂x ∂x c ∂t c ∂t
 2
∂ S ∂2S 1 ∂ 2S

= + 2 − 2 2 ey = 0,
∂x2 ∂z c ∂t
Try solution
S = e−iωt [K cos(αx) + L sin(αx)][N cos(βz) + M sin(βz)].
This satisfies the wave equation provided
1
−α2 − β 2 S = 2 −ω 2 S.
 
c
The electric field is

E = βe−iωt [K cos(αx) + L sin(αx)][−N sin(βz) + M cos(βz)]ex


− αe−iωt [−K sin(αx) + L cos(αx)][N cos(βz) + M sin(βz)]ez .

The tangential component of the field has to vanish at the planes z = 0, b or x = 0, a. This
requires

Ex = 0 at z=0: βM = 0,
Ex = 0 at z=b: βN sin βb = 0,
Ez = 0 at x=0: αL = 0,
Ez = 0 at x=a: αL sin αa = 0,

So we need
sπ rπ
M = L = 0, β= , α= with r, s integer.
b a
3H Electromagnetism 2008-2009 Solutions 4 – mostly Faraday and Maxwell page 14

This covers the special case if α = 0 or β = 0 as well. So the electric field is, up to overall
normalisation,
E = βe−iωt cos(αx) sin(βz)ex − αe−iωt sin(αx) cos(βz)ez ,
where  ω 2 rπ sπ
= α2 + β 2 , α= β= , r, s ∈ Z.
c a b
At least one of r, s has to be non-zero for a non-vanishing electric field. The magnetic field is
ω
B = i e−iωt cos(αx) cos(βz)ey .
c
Remark: This represents an electromagnetic field propagating in a rectangular wave guide.

59 Write out the components of Fαβ , the field strength tensor with lowered indices, and compute
the Lorentz scalar Fαβ Fαβ in terms of E and B . Deduce that if an electromagnetic wave is
circularly polarised in one frame, it should be circularly polarised in all frames.

Solution: Fαβ , as given on the formula sheet is


 
0 −E1 −E2 −E3
 E1 0 −B3 B2 
Fαβ = 
 E2 B3
;
0 −B1 
E3 −B2 B1 0

lowering indices gives  


0 E1 E2 E3
 −E1 0 −B3 B2 
Fαβ =
 −E2 B3
.
0 −B1 
−E3 −B2 B1 0
Multiplying components together and adding,

Fαβ Fαβ = 2 |B|2 − |E|2 .




This quantity is a Lorentz scalar (there are no free indices left over) and so is the same in all
frames. To do the final part, note that light is circularly polarised in a given frame if |B|2 = |E|2
at all times. This is the same as Fαβ Fαβ = 0, and so the condition is preserved when we switch
to any other frame.

You might also like